Q22

 
lesterwhchan
Thanks Received: 0
Vinny Gambini
Vinny Gambini
 
Posts: 1
Joined: June 07th, 2013
 
 
 

Q22

by lesterwhchan Wed Aug 07, 2013 11:16 pm

How can this question be answered without the frames?
User avatar
 
ManhattanPrepLSAT1
Thanks Received: 1909
Atticus Finch
Atticus Finch
 
Posts: 2851
Joined: October 07th, 2009
 
 
 

Re: Q22

by ManhattanPrepLSAT1 Fri Aug 09, 2013 1:56 pm

Unfortunately, there's no quick way to do a question like #22 without frames. You could use your previous work to see if you could eliminate some answers that way, or you'll just have to go into the answer choices to see if any of them could be false--if so, eliminate them.

Here's a hypothetical that shows that answer choice (A) could be false.

Image

Here's a hypothetical that shows that answer choice (B) could be false.

Image

When we go to put L somewhere faster than 5th, we realize that to do so, would not permit us to place either F or G first.

Image

It takes much longer without frames!